Mean Girls "The Limit Does Not Exist" Math Problem

Поділитися
Вставка
  • Опубліковано 19 вер 2022
  • 🌏 Get an exclusive NordVPN offer here ➼ nordvpn.com/michaelpenn
    With the 30-day money-back guarantee, it's risk-free!✌
    🌟Support the channel🌟
    Patreon: / michaelpennmath
    Merch: teespring.com/stores/michael-...
    My amazon shop: www.amazon.com/shop/michaelpenn
    🟢 Discord: / discord
    🌟my other channels🌟
    Course videos: / @mathmajor
    non-math podcast: / @thepennpavpodcast7878
    🌟My Links🌟
    Personal Website: www.michael-penn.net
    Instagram: / melp2718
    Randolph College Math: www.randolphcollege.edu/mathem...
    Research Gate profile: www.researchgate.net/profile/...
    Google Scholar profile: scholar.google.com/citations?...
    🌟How I make Thumbnails🌟
    Canva: partner.canva.com/c/3036853/6...
    Color Pallet: coolors.co/?ref=61d217df7d705...
    🌟Suggest a problem🌟
    forms.gle/ea7Pw7HcKePGB4my5

КОМЕНТАРІ • 184

  • @MichaelPennMath
    @MichaelPennMath  Рік тому +26

    🌏 Get an exclusive NordVPN offer here ➼ nordvpn.com/michaelpenn
    With the 30-day money-back guarantee, it's risk-free!✌

  • @MathNerd1729
    @MathNerd1729 Рік тому +405

    I'm glad you looked at the one-sided limits individually. Way too many UA-cam channels just stop after misunderstanding L'Hôpital and claiming the original limit cannot exist because the new limit doesn't exist.
    For those unaware why that's wrong:
    It's because that line of reasoning doesn't apply for when the new limit oscillates like (x + sin(x))/x as x tends to infinity.
    The actual limit in my example is 1, but applying the other UA-camrs' logic would make it seem like that limit doesn't exist.
    L'Hôpital only guarantees equality when the new limit exists. So, if the new limit doesn't exist, you have to find a different approach or break it up into two limits like what Michael Penn has done! :)

    • @alxlg
      @alxlg Рік тому +3

      I think the actual name is de l'Hôpital

    • @RizmaYudatama
      @RizmaYudatama Рік тому +28

      I believe you can't use L'Hopital rule if your initial limit does not go to 0/0 or infinity/infinity

    • @Dth091
      @Dth091 Рік тому +13

      @@alxlg "de" just means "of", like "William of Orange". l'Hopital is fine to use as a family name to refer to the guy :)

    • @user-yv1qs7sy9d
      @user-yv1qs7sy9d Рік тому +5

      Can you please explain how looking into the side limits solves the problem in l'Hospital's theorem? Specifically, how does checking the side limits of the new limit prove that the original limits doesn't exist?

    • @MichaelRothwell1
      @MichaelRothwell1 Рік тому +9

      @@user-yv1qs7sy9d because if the new expression tends to ∞ (or -∞) then de l'Hôpital tells you that the original expression does the same. In other words, you can apply de l'Hôpital if the new expression converges to a finite limit or diverges to ∞ or diverges to -∞.

  • @johnchessant3012
    @johnchessant3012 Рік тому +9

    such an easy problem for the sudden death final question of a math competition

  • @kevinsong6329
    @kevinsong6329 Рік тому +17

    I watched this movie for the first time at a science summer program. Everyone demanded the movie be paused and solved it 😂

  • @groupforrest456
    @groupforrest456 Рік тому +3

    This is, of course, the most fetch of any maths problems

  • @QuantumHistorian
    @QuantumHistorian Рік тому +370

    The "fast and loose" version is about 10x more rigorous than how any physicist uses Taylor expansions for this kind of problem in practice lol. Which is to only calculate lowest order for the numerator and denominator and see if the lowest powers cancel out or not. I even found writing out the Taylor series as an infinite sum incredibly jarring: I almost always think of the Taylor series as first two or three terms only.

    • @iooooooo1
      @iooooooo1 Рік тому +22

      Another way to write just the low-order terms is to write it as f(x) = a_0 + a_1 x + a_2 x^2 + O(x^3) (assuming 2nd order). Still avoiding the infinite sum notation, but this version is technically correct. Or you could just write Err(x) for the error term, that's also something I've seen done!

    • @KitagumaIgen
      @KitagumaIgen Рік тому +30

      Shush, I thought we all had agreed to keep that a secret? The mathematicians can never know!

    • @christianorlandosilvaforer3451
      @christianorlandosilvaforer3451 Рік тому +1

      thats a little bit disrespectful

    • @pandabearguy1
      @pandabearguy1 Рік тому +10

      Every infinite series has to be written out all infinity terms to be properly checked. There is absolutely nothing rigorous about assuming anything in this world. If you havent counted to infinity, then you can not by any means whatsoever claim that there are infinitely many numbers. This is logic 101

    • @brandonbennett4970
      @brandonbennett4970 Рік тому +32

      @@pandabearguy1 i can see you are new to mathematics. Maybe pick up a book before asserting such ridiculous a statement.

  • @CommanderdMtllca
    @CommanderdMtllca Рік тому +9

    Oh my god Michael, you can't just ask why limits don't exist

  • @bryantwiltrout5492
    @bryantwiltrout5492 Рік тому +49

    Ngl, I actually was able to follow along with this. Watching videos like these has definitely helped with me remembering all my calculus studies. It’s been far too long since I took calculus classes.

  • @9WEAVER9
    @9WEAVER9 Рік тому +49

    I'm just grateful that Michael Penn exists!

    • @aronbucca6777
      @aronbucca6777 Рік тому +8

      You have to demonstrate his existence and uniqueness before being able to say that!

    • @RexxSchneider
      @RexxSchneider Рік тому +7

      @@aronbucca6777 His uniqueness is a sufficient, but not necessary condition.

  • @urieldaboamorte
    @urieldaboamorte Рік тому +2

    you don't understand how happy I am that this crossover happened

  • @manucitomx
    @manucitomx Рік тому +4

    That was meanly fun.
    Thank you, professor!

  • @chillynight201
    @chillynight201 Рік тому +8

    It's been a while since I've done this kind of maths and for some reason I was never taught about the one-sided limits in this way, and I was also never taught about L'Hospital's rule. I wish I would have known it before because it seems very useful. Anyway thank you for this video, it taught me a lot!

  • @EdwardNoguera
    @EdwardNoguera Рік тому +12

    I remember applying L'Hôpital and one-sided limits back when I took Cal 2 in college for this problem, it made me feel like a genius lol thanks for the great vid!

    • @goodplacetostart4606
      @goodplacetostart4606 Рік тому

      Dude I’m doing this kinda stuff since my eighth grade first semester calc 1 class

  • @pauljarski7590
    @pauljarski7590 Рік тому +3

    You can avoid being “sketchy” by adding remainder terms in little o notation to your finite limit expansions, but that’s rigor for the sake of rigor.

  • @AngryArmadillo
    @AngryArmadillo Рік тому +1

    You’re concision is appreciated

  • @FirstnameLastname-hg5gt
    @FirstnameLastname-hg5gt Рік тому +18

    You did exactly what I am teaching my students not to do. There exist cases where the limit of f'(x)/g'(x) does not exist while the limit of f(x)/g(x) exists. (Althought as the limit in the video is not one of them).
    What the De L' Hospital Theorem says, is that (under the other conditions) if the limit of f'(x)/g'(x) exists then the limits of f(x)/g(x) exists. (But the inverse is not in general true). On the other hand De L' Hospital Theorem applies also in side limits (actually this is the method that the theorem is proved), so
    in the case of the video, we may apply the De L' Hospital theorem to the side limits, and in the end arrive to the conclusion that they are different therefore the initial limit does not exist.

  • @goodplacetostop2973
    @goodplacetostop2973 Рік тому +14

    10:41

  • @markmorgan5568
    @markmorgan5568 Рік тому +4

    My mathematics education is sadly lacking, and though I had read about l’Hopital’s rule, I never really got it until ~1:45 of this video. Totally just clicked.
    Thanks!

  • @mameemia
    @mameemia Рік тому

    OMG I TOTALLY NOTICED THIS DURING QATCHING ILYSM

  • @Telogor
    @Telogor Рік тому +1

    I played it even more fast and loose and just did the one-sided limits for the original problem. From the right, we have a very small negative number in the numerator and an even smaller positive number in the denominator, so it's negative infinity. From the left, it's a similar situation, but with a positive numerator, so positive infinity.

  • @sternmg
    @sternmg Рік тому +21

    Having +sin _x_ instead of -sin _x_ in the numerator would've made for a more interesting problem, as the linear _x_ terms in the Taylor series would cancel.

    • @martinepstein9826
      @martinepstein9826 Рік тому +2

      The first time I watched Mean Girls my friend paused with this limit on screen and asked me if I could solve it. I got it wrong because I assumed the linear terms cancelled :(

  • @ddognine
    @ddognine Рік тому +4

    I took the lazy approach since the limit can be broken up into a -csc(x) term which clearly diverges at zero, so the limit doesn't exist. Done. Maybe in certain scenarios you have to worry about additional terms, but it has been a long time since my calc II days. So I will leave it to someone else to correct me if I am wrong!

  • @Sg190th
    @Sg190th Рік тому

    Another math channel doing this problem is wonderful.

  • @jonathonwickathon3163
    @jonathonwickathon3163 Рік тому

    Can finally solve a Michael Penn problem in my head 😭 persistence for the win

  • @AGryphonTamer
    @AGryphonTamer Рік тому +1

    You are the exact spitting image of Tom Scott.
    Trademarked red shirt and everything.

  • @germainfombin9733
    @germainfombin9733 Рік тому

    Nice explanation 👌

  • @blarblablarblar
    @blarblablarblar 28 днів тому

    As Taylor approached Sine of X, he asked the function, " Can you be approximated because you're differentiable? Or are you differentiable because you can be approximated?" He then proceeded to use his cursed technique, "Infinite Differentiation," to try to reduce the function to zero. However the function Sine of X is cyclo-differentiable, and so by contradiction the function stated, "Stand proud, you're strong. But nah, I'd win."
    Brook Taylor fell to his knees. In his dying moments, he uttered the words...
    "Domain Expansion."
    Because when it comes to differentiable functions, always bet on Taylor. The ones who founded calculus, the ones who formalized the derivative, they would all bear witness to this penultimate technique:
    "Infinite Series Expansion!"
    With this, Taylor stood up and said, "Throughout the heavens and the earth, I alone can differentiate."
    -/watch?v=Ow6Y-g4nRls

  • @edwardlulofs444
    @edwardlulofs444 Рік тому

    Instructive, thanks.

  • @RexxSchneider
    @RexxSchneider Рік тому +19

    An even sketchier way of looking at the problem from about 1:26 is to say that ln(1-x) - sinx tends toward 0 - 0 = 0, while the denominator tends toward 0 x 0 = 0² which is a "higher order" of zero, so we should expect the function to behave like k/x when x is very small. That's enough to suggest to me that the limit as x→0 cannot exist, and you can do that in your head.
    Doing some kind of "hand-wavy" analysis has always served me well before proceeding to invest time in more rigorous investigation, because I'm then surprised if that turns in a different answer. A different answer would motivate me to double-check as I almost always find an error somewhere in the more complex working.

    • @TheEternalVortex42
      @TheEternalVortex42 Рік тому +4

      This is good intuition for why the limit DNE

    • @lavneetjanagal
      @lavneetjanagal Рік тому +1

      What do you think about (Sin(x) -x)/x^2 as x -> 0 ? Is 0 - 0 =0 now or is it 0^3?
      This is a very dangerously wrong reasoning you presented.

    • @RexxSchneider
      @RexxSchneider Рік тому +1

      @@lavneetjanagal No, of course not. it should be obvious that the numerator is of the order of x^3 and the denominator is of the order of x^2. The function will look like x for very small values of x and the limit will exist. It's not that difficult (or dangerous!) to do the approximation in your head as I suggested.
      To check the approximation, take the Maclaurin expansion you'll see that the first term of (sin(x) -x) is -(x^3)/6, and so the function you presented gets closer to -x/6 when x is in the neighbourhood of zero.
      [Edit] I can see that I made the mistake in my original post of writing "0 - 0 = 0", when I actually meant "- 0 - 0 = 0" since ln(1-x) is negative for small positive x. Apologies for confusing you.

    • @yvltc
      @yvltc Рік тому

      While this sort of reasoning usually works, there are cases where it fails - spectacularly so. I remember 1st year of university, Calculus I, my professor gave us a very specific limit which I can't recall but I can try to look it up later. My intuitive reasoning similar to yours convinced me the limit was 0, I even checked with a calculator for extremely small values of x (a tactic I used to use whenever I had a limit or summation or whatever and the worksheet didn't have the solution) and it approached 0. Time for the rigorous proof and voilà, the limit was actually 1/24, which isn't even that small of a number for a calculator to "confuse" with 0. I talked to my professor about it and in the next class he said there had been "doubts" regarding said limit, then went on to prove the limit was 1/24 and say calculators make rounding errors that sometimes yield incorrect results like in this case.

    • @RexxSchneider
      @RexxSchneider Рік тому

      @@yvltc Yes, indeed. If my suggested "in the head" way of estimating these sort of limits always gave the right answer, there would be no need for a rigorous derivation of the limit. The point is that having an intuitive idea of what the limit ought to look like acts as a sanity check when we perform the real calculations and helps us spot when we have probably made a mistake in that calculation, so prompts us to recheck our work. I think your anecdote is a good example of that process at work.

  • @self8ting
    @self8ting Рік тому +13

    The second method is not sketchy, that's the method we use in France instead of l'Hopital's rule. It's called "developpement limite" fr.wikipedia.org/wiki/D%C3%A9veloppement_limit%C3%A9 (it's only in French sadly). There's just some notations that are missing in Michael's example for it to be rigorous !

    • @riccardofiori828
      @riccardofiori828 Рік тому +5

      Yess! Also in Italy we use this method with the little o notation, we call it Peano remainder and it's perfectly rigurous and justified by theorems.

    • @samueljele
      @samueljele Рік тому

      I don't think this has something to Do with the country you are from xD everyone uses this. I'd evem say that, since L'Hopital was french, you also use that rule in france ;)

    • @self8ting
      @self8ting Рік тому +2

      @@samueljele That's what's funny ! I've never heard about l'Hopital before watching American Maths videos ahah.

  • @ojas3464
    @ojas3464 Рік тому

    Like the concluding "Good place to stop" Short videos are sensitive to attention span, retention capacity of viewers on challenging at the same time entertaining subjects. Thank you☺

    • @iooooooo1
      @iooooooo1 Рік тому +1

      Agreed. He does this for all his videos, there's even a novelty account @Good Place To Stop that points out the timestamp where he says it.

    • @ojas3464
      @ojas3464 Рік тому

      @@iooooooo1 Indeed! I've seen folks stop eating while still enjoying, Trying to saturate oneself maybe inadvisable in some situations!☺

  • @Minecraft2331
    @Minecraft2331 Рік тому

    Fast and loose works if you can demonstrate that for the expansion everything to the right of the leading terms don't affect the limit, which seems pretty doable

  • @davidmitchell3881
    @davidmitchell3881 Рік тому +2

    Minor simplification. 2 sin x cos x is equal to sin 2x. This eliminates the need to worry about the behaviour of cos x as x tends to zero.
    Its a trival thing but it eases the cognitive load a tad which is alawys a good thing

  • @Albeit_Jordan
    @Albeit_Jordan Рік тому +2

    so if I was in Lindsey Lohan's shoes, i would have won because I answered "The limit does not exist" but if I was asked for proof I would have failed because I would have just written "it's undefined/a singularity"

    • @radeklew1
      @radeklew1 Рік тому

      What about the limit as x goes to zero of x/x? :)

  • @holyshit922
    @holyshit922 Рік тому +2

    One sided limits can be calculated without L'Hospital rule and Taylor expansion (both need derivatives)
    We will get the limits which appears in derivatives of ln , and trig functions such as sin , cos , tan
    but we can calculathe this one sided limits without derivatives

  • @engjayah
    @engjayah Рік тому

    we can apply the result sinx/x = 1 as x approaches zero directly to simplify the expression by dividing both numerator and denominator by x^2 from which the expression reduces to ln(1-x)/x^2 - 1/x;
    Now by applying Taylor series expansion -(x+x2/2++x3/3+......)x2 -1/x expression reduces to (-1/2-2/x) as x approaches to zero from which it is clear that the limit is not existing.

  • @af9466
    @af9466 Рік тому +43

    A non-sketchy way to do it with Taylor series would be to write the first n terms of the expansion plus o(x^n), in asymptotic notation. Then, after dividing the numerator and the denominator by x^n, you'd get o(x^n)/x^n which cancels out to 0 as x approaches x0 (0 here) - by the o property, and the remaining terms would give you the limit in a usual way.
    Note: f(x) = o(g(x)), x->x0 means for all e>0, there exists d>0 s.t. for all x in d-vicinity of x0, excluding x0, intersected with f's domain, we have |f(x)| < e|g(x)|.
    If g is nonzero anywhere at that vicinity with x0 excluded, and has the same domain as f, then this is equivalent to lim(f(x)/g(x))=0, x->x0.

    • @hrishikeshpatel6670
      @hrishikeshpatel6670 Рік тому

      Jee student??

    • @af9466
      @af9466 Рік тому

      @@hrishikeshpatel6670 Hi! Did you mean JEE in India?
      (I'm not-I'm a student from Ukraine.)

  • @hrishikeshpatel6670
    @hrishikeshpatel6670 Рік тому +1

    Le jee students* we eat sums li this in our breakfasts😂
    Jee advance be like we didn’t even consider this as a problem

  • @mtaur4113
    @mtaur4113 Рік тому

    The Taylor approximations aren't cut at the same places, but second order + O(x^3) should be sufficient to get something rigorous here. In particular, x^3*convergent is O(x^3), and x*O(x^3)->0 as x->0.

  • @haakoflo
    @haakoflo Рік тому +1

    As x->0, sin(x)->x (ie sin(x)/x -> 1), so the limit will be the same as for (ln(1-x) - x)/x^2. It's pretty easy to visualize that it approaces negative infinity for x->0(+) and positive infinity for x->0(-)

  • @Sumit-df5nn
    @Sumit-df5nn Рік тому +1

    Him : Lopitals rule
    Jee Aspirants: L Hospital🏥🚑 rule

  • @ichigo_nyanko
    @ichigo_nyanko Рік тому

    Could you do a video of the proof of l'Hospital?

  • @michalchik
    @michalchik Рік тому +1

    I just split it into two fractions, the right fraction is negative sign over sine squared, which reduces to negative one over sine, which as X approaches 0 becomes-1/0 which is undefined therefore the limit is undefined

    • @tc14hd23
      @tc14hd23 Рік тому

      I'm afraid this logic doesn't work. Just because there is an undefined expression like -1/0, doesn't mean that the limit is undefined. By that same logic the limit x/x as x approaches 0 would also be undefined, but it's 1.

    • @michalchik
      @michalchik Рік тому

      @@tc14hd23 interesting point but I don't think this is relevant because this is much more like the limit as X goes to zero of one over X. In the limit you're taking there's a cancellation for all numbers that are not zero, there's no cancellation in the number flies off towards Infinity and negative Infinity

  • @greymatter7547
    @greymatter7547 Рік тому

    Very good explanations, nice reminders of these calculus ideas. So many videos assume you still remember all of these things

  • @scipionedelferro
    @scipionedelferro 7 місяців тому

    No need to use l'Hopital or Taylor or even to know derivatives. Just factorize 1/sinx and divide by x numerator and denominator : [ ln(1-x)/x - sinx/x] [ sinx/x] -> -2 (all well known elementary limits). Then we're left with 1/sinx, whose limit to 0 does not exist obviously.

  • @samueldevulder
    @samueldevulder Рік тому +1

    This isn’t sketchy provided you use the little o(x^4) notation.

  • @andraspongracz5996
    @andraspongracz5996 Рік тому +1

    A much nicer problem would be lim (ln(1-x)+sin(x))/(1-cos(x)). The Taylor expansions yield the correct result -1.

  • @HiiImChris
    @HiiImChris Рік тому

    No big issue or anything but for some reason the l'hospitals in the board was killing me lol.

  • @numberandfacts6174
    @numberandfacts6174 Рік тому +1

    Riemann hypothesis solutions

  • @davidmuqattash9811
    @davidmuqattash9811 Рік тому +1

    I may be seeing this incorrectly, but didn't he forget to add the negative sign when taking the derivative of ln(1-x)?
    It should be 1/(1-x) × (-1)
    Nevermind he factored that into the denominator lol.

  • @pandabearguy1
    @pandabearguy1 Рік тому

    If you set yourself limits in life you will forever be limited

  • @rusparmesan
    @rusparmesan Рік тому

    Isn't L'Hopital rule essentially just a simplified version of Taylor in this case? You get Taylor series by taking derivatives N times. As for "fast and loose", maybe you could've used o(x^n) remainder to be slightly more accurate with it.

  • @sergiolucas38
    @sergiolucas38 Рік тому

    good video :)

  • @physicorum7107
    @physicorum7107 Рік тому +1

    Multiverse of madness

  • @alnitaka
    @alnitaka Рік тому

    I wonder where Carolyn Kraft in the movie came up with the answer -1.

  • @aqdjbcr
    @aqdjbcr Рік тому

    Is the first step required or can you just plug in and realize cos0 is 1 to get lhospitals

  • @_judge_me_not
    @_judge_me_not Рік тому

    You can try problems from ISI (Indian Statistical Institute) B.Math and M.Math entrance exams ...
    or even Chennai Mathematical Institute (CMI) entrance questions
    they are brilliant

  • @debunkosaurus8228
    @debunkosaurus8228 Рік тому

    The simpler way to do this is to recognize that, for small x, sin(x)->x. This makes the equation become (ln(1-x)/x^2)-(1/x). From this, its obvious that the limit is undefined.

  • @GreRe9
    @GreRe9 Рік тому

    Isn't the taylor expention and l'Hôptial rule doing basicly the same here lim u/v = lim u' *x/v' *x = lim u'/v'

  • @JakubH
    @JakubH Рік тому

    If the limit after l'Hospital turns out to be nonexistent, you cannot say, that the original limit is nonexistent...
    In this case it works - is it because there is a non-said default implied "I was actually doing the side limits from the beginning, I just didn't bother writing it separately"? Like, if you did it rigorously, you find the two side limits separately from the beginning using l'Hospital, they turn out -inf and inf. Realize the side limits differ, so the actual limit does not exist. Is this the case? To me it makes sense this way. Suddenly there is no problem with the l'Hospital and non-existent limit.

  • @MurshidIslam
    @MurshidIslam Рік тому

    2:53 What is the difference between "the limit does not exist" and "the limit is infinite"?

  • @mikesteele5935
    @mikesteele5935 Рік тому

    It's L'Hopital's rule that is sketchy ... the three term O approximations simply tell us the truth !

  • @EnPee91
    @EnPee91 Рік тому

    L'hospital's rule? Lol

  • @porcorosso4330
    @porcorosso4330 Рік тому

    I guess the small angle approximation will also yeild similar results.
    I guess it is the same as using the Taylor series.
    ( I took a antenna class in college.
    It is basically the solution to the entire class.
    When you can't solve a problem by hand, just use the small angle approximation.
    Congratulations, you passed the class.)

    • @deept3215
      @deept3215 Рік тому

      Small angle approximation is a first order approximation, so whenever you need higher order expansion it won't work

    • @porcorosso4330
      @porcorosso4330 Рік тому

      @@deept3215
      In this case, it will work since we are taking the limit to 0.
      All the other terms will be much smaller than the 1st order term.
      Obviously, it will not work if the angles are not small.

    • @deept3215
      @deept3215 Рік тому

      @@porcorosso4330 That's not how it works. As I said small angle approximation is a first order approximation. Now consider for example lim of (x-sin x)/x^3 as x approaches zero. Small angle approximation would say that's zero. Is that right? What about (x-sin x)/x^4 ?

    • @porcorosso4330
      @porcorosso4330 Рік тому

      @@deept3215
      No, in your example we should look beyond the 1st term.
      For these 2 example, it should be enough to expand to the 2nd term.

    • @deept3215
      @deept3215 Рік тому

      @@porcorosso4330 And that's exactly what "small angle approximation is a first order approximation" means. And why it doesn't "always" work.

  • @leickrobinson5186
    @leickrobinson5186 Рік тому

    Just doing this quickly in my head, but doesn’t it just approach -2/x as x → 0?

    • @RexxSchneider
      @RexxSchneider Рік тому

      That's quite right, and IMHO it's good to get an idea in your head of what to expect if you were to do a more rigorous investigation.

  • @danielmilyutin9914
    @danielmilyutin9914 Рік тому

    One can use equivalence rules. sin x is x. ln(1-x) is -x. cos(x) is 1 - x²/2
    fix misprint on cos(x)

    • @bjornfeuerbacher5514
      @bjornfeuerbacher5514 Рік тому +2

      Huh? I've never heard these called "equivalence rules". These are essentially the first terms of the Taylor expansions. And that's essentially what Michael actually _did_ do in the second part of the video.
      And BTW, "cos(1-x) is 1 - x²/2" is simply wrong. You have not only one, but several errors / typos in that claim...

    • @RexxSchneider
      @RexxSchneider Рік тому +1

      You're correct that for small x, sin(x) ≈ x and ln(1-x) ≈ - x, but for small x, we find that 1 - cos²(x) = sin²(x) ≈ x².

    • @patricius6378
      @patricius6378 Рік тому +3

      I'd like to demystify "equivalence rules". It's called the fundamental theorem of engineering. You're welcome.

    • @danielmilyutin9914
      @danielmilyutin9914 Рік тому

      @@RexxSchneider I see no contradoction here.
      denominator is best turned to sin²(x) ≈ x².
      I meant this.
      What I meant was go as simple as possible. Not to use overkill. Like use L`Hopital`s rule or Taylor series.
      Just note. One can use 1 - (1 - x²/2 )^2 as well. it will also turn to x^2.
      Note misprint fix in first comment.

    • @danielmilyutin9914
      @danielmilyutin9914 Рік тому

      @@patricius6378 Equivalence rules is standard tool for limits. No mystics in it. So I don't get your point.

  • @Fred-yq3fs
    @Fred-yq3fs Рік тому

    Use limited developments of order 1 in x and done in 1 min!
    ln(1-x) = -x (could integrate the geometric series 1+x+x2+... = 1/(1-x))
    sin(x) = x
    1-cos(x)^2 = sin(x)^2 = x^2
    Then lim = -2x/x^2 = -2/x
    x->0+ => lim = -inf
    x->0- => lim = +inf

  • @TrustTheScience
    @TrustTheScience Рік тому

    When would you need to know this in real life, outside of teaching it, or taking a course on it?

  • @robert-skibelo
    @robert-skibelo Рік тому

    I enjoyed this video, even though I have never heard of the Mean Girls.
    The man's name was L'Hôpital, not L'Hospital, as your correct pronunciation suggests you know. Because the circumflex over the o originally indicated that an s had been left out, I suppose you could argue that you haven't made a mistake, but simply used an antiquated spelling that you saw in some old document somewhere.

    • @reamick
      @reamick Рік тому

      The marquis himself spelled it L'Hospital.

    • @robert-skibelo
      @robert-skibelo Рік тому

      @@reamick Thanks. I'd be genuinely interested to know how you know this. Have you seen manuscripts written by the marquis himself? If not, the "Remarque orthographique" on the French wikipedia page observes that at least one scholar closely associated with the marquis used the spelling with circumflex. I wouldn't put too much store on printed title pages of the period: printers had their own style conventions that they would have followed when two spellings were regarded as equally valid. Though none of this is quite as interesting as the question of why Professor Penn chose to depart from the spelling most commonly accepted by today's mathematicians.

  • @ivanvz
    @ivanvz Рік тому

    could you say that the limit is undefined

  • @puscjom8279
    @puscjom8279 Рік тому +1

    2:24 but derivarive of ln(1-x) is -(1/(1-x))

    • @AlexK-jp9nc
      @AlexK-jp9nc Рік тому

      -(1-x)=x-1
      Note that he mentions that he's changing the order of subtraction when he writes it

  • @sphenisc3
    @sphenisc3 Рік тому

    That was so fetch.

  • @igglye6805
    @igglye6805 Рік тому

    I thought L’Hopital was taking the derivative of the top AND bottom

  • @txikitofandango
    @txikitofandango Рік тому

    I'm a be honest with you, I probably would've missed turning 1 - cos^2 into sin^2

  • @Sumit-zg3tj
    @Sumit-zg3tj Рік тому

    The guy looks like a very intelligent version of Forrest Gump

  • @comma_thingy
    @comma_thingy Рік тому

    lim of numerator is 1, while limit of denominator is 0 and since the denominator is always positive in a nbhd of 0 (not containing 0), this implies the limit is infty.
    Is this not enough?

    • @thesecondderivative8967
      @thesecondderivative8967 Рік тому +1

      sin x in the numerator will be positive from the right and negative from the left. Therefore, the numerator is negative from the right and positive from the left. This means the Lim as x approaches 0+ is negative infinity and the limit as x approaches 0- is +inf. This means the limit does not exist. Also, the limit of the numerator is 0 not 1.

    • @comma_thingy
      @comma_thingy Рік тому

      @@thesecondderivative8967 yeh i had a bit of a brain fart and did ln(1) = 1

    • @RexxSchneider
      @RexxSchneider Рік тому +1

      The limit of the numerator is 0. Check that for small x, we find ln(1-x) ≈ - x and sin(x) ≈ x. So for small x, the function looks like (-x - x) / (x²) ≈ - 2/x which has no limit at x=0 (it's both +infinity and -infinity).

  • @BarryRowlingsonBaz
    @BarryRowlingsonBaz Рік тому +2

    I've not seen the movie so to save anyone else having to look for it its in this clip here: ua-cam.com/video/EligNcjdyyI/v-deo.html

  • @orenfivel6247
    @orenfivel6247 Рік тому

    is it a problem to conclude during L'hospital application that if lim(f'(x)/g'(x);x->a) DNE then lim(f(x)/g(x);x->a) DNE?

    • @samueljele
      @samueljele Рік тому +1

      Remember that L'Hopital is formulated as the equality of onesided limits of f/g and f'/g'.
      So if the limit from below or above of f'/g' exists in |R-bar (i.e. |R with +- infinity), then you can conclude that the same is true for f/g (from the same direction). That means if both, the limit from above and below, exist in |R-bar, but are not the same (e.g. one is +infinity and the other is -infinity, as in this example), then both the limit of f'/g' and the limit of f/g do not exist. But if the limit from above or below of f'/g' does not exist because it is for example oscillating, you can't conclude the behaviour of f/g.
      I'm not 100% sure, so please someone correct me if I'm wrong.

    • @orenfivel6247
      @orenfivel6247 Рік тому

      @@samueljele i think U can use L'Hopital in this case if u cacluate the original limit from each direction, and then apply L'Hopital for each direction ie:
      L⁺=Lim{f'(x)/g'(x);x→0⁺}=-∞ ⇒ Lim{f(x)/g(x);x→0⁺}=L⁺=-∞
      L⁻=Lim{f'(x)/g'(x);x→0⁻}=+∞ ⇒ Lim{f(x)/g(x);x→0⁻}=L⁻=+∞
      L⁺≠ L⁻⇒ Lim{f(x)/g(x);x→0} DNE

    • @orenfivel6247
      @orenfivel6247 Рік тому

      @@samueljele observe non ex.:
      f=x^2⋅sin(1/x)
      g=sin(x)
      f'/g'=(2x⋅sin(1/x) + cos(1/x))/(cos(x))
      lim(f'/g';x→0) DNE b/c lim(cos(1/x);x→0)
      but as x→0, x/sin(x)→1 and thus x^2/sin(x)→0,
      and since sin(1/x) is bounded between -1 and 1, we have
      lim(f/g;x→0)=lim([x^2/sin(x)]⋅[ sin(1/x)];x→0)=0,,

    • @samueljele
      @samueljele Рік тому

      @@orenfivel6247 yes exactly that's what I meant.

  • @sergiocaprara1314
    @sergiocaprara1314 Рік тому +1

    Your Taylor series for the numerator is not correct because you kept x^3 terms coming from sin(x) but you neglected same order term coming from log(1-x). Of course, this is immaterial when taking the limit, but I discourage my students from writing incoherent Taylor expansions, meaning expansions where different contributions are expanded to different order in x

  • @martinalistairfip7204
    @martinalistairfip7204 Рік тому

    Fun fact: L'Hôpital's rule wasn't actually invented by l'Hôpital but by Johann Bernoulli who then sold it to l'Hôpital

    • @Alex_Deam
      @Alex_Deam Рік тому

      Maybe Michael could make purchasing Fermat's Last Theorem from Andrew Wiles a tier on his patreon

  • @mikispo1
    @mikispo1 Рік тому

    Or just (-x+o(x)-x+o(x))/(x+o(x))² = (-2x+o(x))/x²= -2/x (thus the limit at x →0 doesn't exist in ℝ ∪ {-∞, +∞} with the usual topology).

  • @1SingleT
    @1SingleT Рік тому

    I have no idea what any of this means

  • @Elijah__Don
    @Elijah__Don Рік тому +2

    Be sure to note that the reason this works is because the one sided limits have the form #/0, meaning an infinitely small pos number is basically inf and an infinitely neg number is neg inf. Otherwise, this logic would imply that a straight line has a DNE limit everywhere.

  • @arekkrolak6320
    @arekkrolak6320 Рік тому

    the only tricky thing in this problem is the proof of the L'Hospital rule :)

  • @ved9402
    @ved9402 Рік тому +2

    Jee aspirants be like are you comedy me 🤣🤣🤣🤣

  • @tbg-brawlstars
    @tbg-brawlstars Рік тому

    Why am I getting -1/2 after using L'hopitals rule

  • @karmaWh
    @karmaWh Рік тому

    Can't you just consider the limit of 1/sin, which blows up the whole thing?

    • @edvink8766
      @edvink8766 Рік тому +1

      because lim (A + B) = limA + limB if and only if limA and limB exists. Because the limit of 1/sin doesn't exist, you can't just isolate that term.

  • @T1MB05L1C3
    @T1MB05L1C3 Рік тому

    1:24 wait... the world didn't blow up?

  • @taleladar
    @taleladar Рік тому

    I wonder when we analyze the limits, why we cannot just say the limit can be EITHER a positive or a negative infinite value. In other words, instead of saying a solution doesn't exist, why can't we say two distinct solutions exist?

    • @danielleza908
      @danielleza908 Рік тому

      Because that's not the definition of a limit. Loosely speaking, if a limit exists, all of the functions values should "tend towards it" as x approaches 0.
      That said, the one sided limits do exist.

  • @aureusvisions
    @aureusvisions Рік тому

    Am i the only one that watch these types of math videos without understanding them? I'm in 8th grade and i have no idea of what this guy is talking about

  • @theartisticactuary
    @theartisticactuary Рік тому

    9:14 Your x cubed term is wrong. You've taken x cubed term from sin but ignored x cubed term from log. I know x cubed term doesn't matter in the end but this has to lose you a mark.

  • @nychan2939
    @nychan2939 Рік тому

    Maybe, +∞ = -∞ = ∞

  • @DDG00
    @DDG00 10 місяців тому

    Huh?

  • @sagarmajumder7806
    @sagarmajumder7806 Рік тому

    Sir, when I saw trig functions like sinx,tanx and limit (xtends to 0).I considered them as x.This is very helpful in exam 😇😇😇😇😅😅

  • @Andres-ww7mq
    @Andres-ww7mq Рік тому

    L'Hôpital Rule and not L'Hospital's Rule. It's a last name so don't change it!

  • @turritopsisrockola
    @turritopsisrockola Рік тому

    I'm sure lindsay lohan could pull it off

  • @user-hq7hi2sl2o
    @user-hq7hi2sl2o Рік тому +1

    asnwer=1 isit 🤣🤣

  • @tomholroyd7519
    @tomholroyd7519 Рік тому

    Rick & Morty

  • @touristofsongs4946
    @touristofsongs4946 Рік тому

    how the fuck is
    ln(1-x)' = 1/(x-1)

  • @mr.twicks3009
    @mr.twicks3009 Рік тому

    Love the videos but absolutely despise how you draw your infinites